LSAT and Law School Admissions Forum

Get expert LSAT preparation and law school admissions advice from PowerScore Test Preparation.

 Administrator
PowerScore Staff
  • PowerScore Staff
  • Posts: 8919
  • Joined: Feb 02, 2011
|
#100908
Complete Question Explanation

The correct answer choice is (E).

Answer choice (A):

Answer choice (B):

Answer choice (C):

Answer choice (D):

Answer choice (E):

This explanation is still in progress. Please post any questions below!
 ChicaRosa
  • Posts: 111
  • Joined: Aug 23, 2016
|
#30683
Is E correct because of ln 10-14?

Thanks!
 Adam Tyson
PowerScore Staff
  • PowerScore Staff
  • Posts: 5153
  • Joined: Apr 14, 2011
|
#31260
I'll say yes, Chica, in part because of those lines, but also in part because of what came before, starting at line 5, because that's where we learn that perfect justice is impossible.

It's important to note, though, that it isn't just about where we can find proof of that answer in passage A. The other half of the question requires that we be cure that we do NOT find support for that choice in passage B. The second passage has to NOT use that principle.

Good work!

Get the most out of your LSAT Prep Plus subscription.

Analyze and track your performance with our Testing and Analytics Package.